in Quantitative Aptitude recategorized by
543 views
0 votes
0 votes

Let $x^2-2(4k-1)x+15k^2-2k-7>0$ for any real value of $x$. Then the integer value of $k$ is

  1. $2$
  2. $4$
  3. $3$
  4. $1$
in Quantitative Aptitude recategorized by
by
543 views

1 Answer

1 vote
1 vote

$x^2 - 2(4k-1)x+15k^2 -2k-7 >0$

$(A)$

putting $k=2$

$x^2 - 6x+49>0$

Here we can't get a negative value since $x^2 +49$ will always be greater than $6x$.  so this is the correct option.


$(B)$

putting $k=4$

$x^2 -30x +225 >0 \implies (x-15)^2 >0$

so on putting $x=15$ we get $0>0$  so this could not be the answer


$(C)$

putting $k=3$

$x^2-22x-122>0$

On putting $x=1$ we are getting $1-22-122>0$ so this could not be the answer

$OR$

$x^{2}-22x-122=x^{2}-22x+121-121-122=(x-11)^{2}-243$ which is negative for any number $x\epsilon[-4,15]$

so this could not be the answer


$(D)$

putting $k=1$

$x^2 - 6x+6>0$

On putting $x=2$ we are getting $4-12+6>0$ so this could not be the answer.

$OR$

$x^{2}-6x+6=x^{2}-6x+9-9+6=(x-3)^{2}-3$ which is negative for $x=4$. so this could not be the answer.

edited by

3 Comments

edited by

nice explanation @Satbir 

it would be better if equation in option c is written as$x^{2}-22x-122=x^{2}-22x+121-121-122=(x-11)^{2}-243$ which is negative for any number x$\epsilon$[-4,15]..Hence false

similarly, equation in option d can be written as $x^{2}-6x+6=x^{2}-6x+9-9+6=(x-3)^{2}-3$ which is negative for x=4.Hence false.

1
1
check your solution for d, it is coming 4-3 = positive.
1
1
thanks.. i have updated it now
0
0

Related questions

Quick search syntax
tags tag:apple
author user:martin
title title:apple
content content:apple
exclude -tag:apple
force match +apple
views views:100
score score:10
answers answers:2
is accepted isaccepted:true
is closed isclosed:true